Good inequality...

This topic has expert replies
User avatar
Legendary Member
Posts: 934
Joined: Tue Nov 09, 2010 5:16 am
Location: AAMCHI MUMBAI LOCAL
Thanked: 63 times
Followed by:14 members

Good inequality...

by [email protected] » Mon May 07, 2012 12:30 am
"If x is negative, is x < -3 ?

(1) x^2 > 9
(2) x^3 < -9 "


I donot have the OA but when I solved the question, I got the answer as E. Is it correct? Could any of the experts please help?
IT IS TIME TO BEAT THE GMAT

LEARNING, APPLICATION AND TIMING IS THE FACT OF GMAT AND LIFE AS WELL... KEEP PLAYING!!!

Whenever you feel that my post really helped you to learn something new, please press on the 'THANK' button.

Legendary Member
Posts: 2789
Joined: Tue Jul 26, 2011 12:19 am
Location: Chennai, India
Thanked: 206 times
Followed by:43 members
GMAT Score:640

by GmatKiss » Mon May 07, 2012 12:56 am
imo: D

Master | Next Rank: 500 Posts
Posts: 142
Joined: Thu Apr 26, 2012 3:24 am
Location: India
Thanked: 28 times

by mathbyvemuri » Mon May 07, 2012 12:59 am
statement(1): x^2 > 9 => x < -3 or x > 3
[spoiler]There is ambiguity here and hence, (1) alone is not sufficient[/spoiler]

statement(2): x^3 < -9 => x < -2.something
[spoiler]There is ambiguity again as it is asked whether x < -3. Hence, (2) alone is not sufficient[/spoiler]

But if we consider both the statements together, [spoiler]the common possible solution is: x < -3, which is sufficient and hence option "C"[/spoiler]

User avatar
Master | Next Rank: 500 Posts
Posts: 385
Joined: Mon Apr 16, 2012 8:40 am
Location: Pune, India
Thanked: 186 times
Followed by:29 members

by aneesh.kg » Mon May 07, 2012 1:15 am
Guys,
the answer is [spoiler](A)[/spoiler].

You can read it here:
https://www.beatthegmat.com/good-inequal ... tml#470202
Aneesh Bangia
GMAT Math Coach
[email protected]

GMATPad:
Facebook Page: https://www.facebook.com/GMATPad

User avatar
GMAT Instructor
Posts: 1248
Joined: Thu Mar 29, 2012 2:57 pm
Location: Everywhere
Thanked: 503 times
Followed by:192 members
GMAT Score:780

by Bill@VeritasPrep » Mon May 07, 2012 7:32 am
mathbyvemuri wrote:statement(1): x^2 > 9 => x < -3 or x > 3
[spoiler]There is ambiguity here and hence, (1) alone is not sufficient[/spoiler]

statement(2): x^3 < -9 => x < -2.something
[spoiler]There is ambiguity again as it is asked whether x < -3. Hence, (2) alone is not sufficient[/spoiler]

But if we consider both the statements together, [spoiler]the common possible solution is: x < -3, which is sufficient and hence option "C"[/spoiler]
Statement 1 works because the stem specifically mentions that x must be negative.
Join Veritas Prep's 2010 Instructor of the Year, Matt Douglas for GMATT Mondays

Visit the Veritas Prep Blog

Try the FREE Veritas Prep Practice Test

Senior | Next Rank: 100 Posts
Posts: 32
Joined: Fri Sep 09, 2011 10:05 am
Thanked: 1 times

by zueswoods » Mon May 07, 2012 8:07 am
I believe the answer is A

The if you draw the graphs you will get the solutions, also, remeber the question stem says X < 0 so X>3 is not an option.